Joan weighs 10 pounds less than her older sister. The average of the two sisters’ weights is 85 pounds. How much does Joan’s older sister weigh?

Answers

Answer 1

Answer:

Joan's older sister weighs 90 pounds

Step-by-step explanation:

x = older sister

x - 10 = Joan

(x + (x-10))/2 = 85

2x - 10 = 170

2x = 180

x = 90

Answer 2

Answer:

90 pounds

Step-by-step explanation:

We can set up an equation to find their weights.

Let's start by naming Joan's weight x.

Her sister's weight would then be x+10, since Joan weighs 10 pounds less.

To find the average between 2 numbers, you need to add them together, then divide by 2.

So we can set up the following equation:

(x+x+10)/2=85

Now let's isolate x.

We can first multiply both sides by 2.

x+x+10=170

Combine like terms.

2x+10=170

Subtract 10 from both sides.

2x=160

Subtract both sides by 2.

x=80

Joan weighs 80 pounds.

x+10 is her sister's weight.

80+10=90

Joan's older sister weighs 90 pounds.


Related Questions

Plz help me this is probably easy but im just not seeing it plz help me ASAP

Answers

Answer: is attatched to the photo I sent

Step-by-step explanation:

I admit, this is tricky, just remember to think about it as where the shapes are the most similar. I'm not exactly sure how to explain it, but where it reflects like a mirror is the "copy"

The example I'm thinking of attached too, but these over lap so its diffucult

4) 101
Is it rational or irrational

Answers

Answer:

rational

Step-by-step explanation:

according to google the definition of RATIONAL is

(of a number, quantity, or expression) expressible, or containing quantities that are expressible, as a ratio of whole numbers. When expressed as a decimal, a rational number has a finite or recurring expansion. Examples of rational numbers are 4 3 and 9

AND THE DEFINITION OF IRRATIONAL IS

(of a number, quantity, or expression) not expressible as a ratio of two integers, and having an infinite and nonrecurring expansion when expressed as a decimal. Examples of irrational numbers are the number π and the square root of 2.

so 101 is a whole number and whole numbers are always rational

HOPE I HELPED

PLS MARK BRAINLIEST  

DESPERATELY TRYING TO LEVEL UP

✌ -ZYLYNN JADE ARDENNE

JUST A RANDOM GIRL WANTING TO HELP PEOPLE!

                       PEACE!

Three people are standing on a Cartesian coordinate plane. Robert is standing at point $(4,3)$, Lucy at point $(6,1)$, and Liz at point $(1,7)$. How many units away is the farther person from Robert?

Answers

Answer: Liz, she is 5 units away from Robbert.

Step-by-step explanation:

Ok, the data that we have is:

Robert is at (4,3 )

Lucy is at (6, 1)

Liz is at (1, 7)

We want to find the units between Robert and the person that is farther away from him.

The distance between Robert and Lucy can be calculated as:

First we calculate the difference in the point or in the position vector:

(4, 3) - (6, 1) = (4 - 6, 3 - 1) = (-2, 2)

To calculate the exact distance, we need to calculate the magnitude of this point, this is:

For a point (x, y), the magnitude is M = √(x^2 + y^2)

Then we have:

D = √( (-2)^2 + 2^2) = √8

Now, the distance between Robert and Liz can be calculated in the same way:

(4, 3) - (1, 7) = (4 - 1, 3 - 7) = (3, -4)

The magnitude is:

D = √(3^2 + (-4)^2) = √(9 + 16) = √25 = 5

Now, we know that 5 > √8

Then the person that is farthest away from Robert is Liz, and she is 5 units away.

the school bought a sandbox that measures 50 meters long 25 meters wide and 5 meters tall how many cubic meters if sand would you need to buy if each cubic meter of sand cost $1.50 how much money would it cost to fill the sandbox

Answers

Answer:

Cost of sandbox = $9,375

Step-by-step explanation:

Given:

Height of sandbox = 5 m

Length of sandbox = 50 m

Width of sandbox = 25 m

Cost of 1 cubic meter = $1.50

Find:

Cost of sandbox

Computation:

Volume of sandbox = (50)(25)(5)

Volume of sandbox = 6,250 m³

Cost of sandbox = 6,250 × $1.50

Cost of sandbox = $9,375

II
Initial Knowledge
This morning, Leila's car had 19.79 gallons of fuel. Now, 2.8 gallons are left. How much fuel did Leila use?

Answers

Answer:

[tex]19.79 - 2.8 = 16.99gallons[/tex]

Compute the range and interquartile range for the data collected for boys and girls. Describe their differences in detail using specific terms of spread. (4 points)

Answers

Answer:

The measure of central tendency, mean and median are approximately equal for the boys indicating that the data of the boys is more evenly spread while standard deviation of the girls data is less than those of the boys indicating that the data for the girls is less widely spread.

Step-by-step explanation:

The given data are;

,             1       2       3      4     5      6       7    8      9      10

Girls,    50    32     15    56   81    50     18    81    22    55

Boys,   75     41     25    22    7     0      43    12    45    70

Sorting the data gives;

Girls,     15,   18,     22,   32,  50,   50,   55,  56,    81,  81

Boys,     0,     7,     12,     22, 25,    41,    43, 45,     70, 75

For the even numbered sample data size, the first quartile, Q₁ is found by sharing the data into two and finding the median of the left half which  gives;

10/2 = 5 on each half

The first quartile, Q₁, is the median of the left 5 data points which is the 3rd data point = 22 for girls and 12 for boys

The third quartile, Q₃, is found in similar method to be the 8th data point which is 56 for girls and 45 for boys

The median = 50 for girls and 33 for boys

Therefore, the interquartile ranges are;

IQR = 56 - 22 = 34 for girls, 45 - 12 = 33 for boys

We check for outliers.

Q₁ - 1.5×IQR = 22 - 1.5*34 = -29

Q₃ + 1.5×IQR = 56 + 1.5*34 = 107

We check the mean of both data samples as follows;

Average for the girls = 46

Average for the boys = 34

Standard deviation for girls = 23.99

Standard deviation for girls = 25.43

Therefore, the measure of central tendency is more accurate for the boys indicating that the data of the boys is more evenly spread while the data for the girls is less widely spread.

-4(-w-10)=
i need help don’t know what the answer is

Answers

Answer:

[tex]\boxed{4w+40}[/tex]

Step-by-step explanation:

Use the distributive property to evaluate the equation.

-4(-w - 10)

4w + 40

-4 * -w = 4w because two negatives cancel to make a positive value.

-4 * -10 = 40 for the same reason.

Each day that a library book is kept past its due date a 30 dollar fee is charged at midnight which ordered pair is a viable solution if x represents the number of days that a library book is late and y represents the total fee

Answers

THIS IS THE COMPLETE QUESTION BELOW;

Each day that a library book is kept past its due date, a $0.30 fee is charged at midnight. Which ordered pair is a viable solution if x represents the number of days that a library book is late and y represents the total fee?

Answers:

(–3, –0.90)

(–2.5, –0.75)

(4.5, 1.35)

(8, 2.40)

Answer

(8, 2.40)

Step by step Explanation

✓We can denote the number of days library book is late = X,

✓We can denote the the total fee = Y.

We were told $0.30 fee is charged at midnight.

Then for lateness for just 1day,the charged fees= 1day ×

$0.30

For X number of days the charged fees= Xday ×$0.30

Therefore, total charge for lateness for X number of days late = Y.

Then can be expressed as

Y= 0.30 * X...............eqn(1)

We can now test the option one after the other

FIRST OPTION (-3, -0.9)

Here we should know the number of days cannot be negative so there is no need of testing in the equation (1)

SECOND OPTION (-2.5, -0.75)

Here we should know the number of days cannot be negative so there is no need of testing in the equation (1)

THIRD OPTION(4.5, 1.35)

here the number of days will definitely be a whole number not 4.5, it's either

charge for 4 days or 5 days.

FORTH OPTION (8, 2.40)

this should be correct because the number of days is whole number and not negative, then if we test it from our equation it satisfy the equation too

Y= 0.30 * X...............eqn(1)

Y= 0.30 * X

2.40= 0.30 * 8

2.40 = 2.40.

Therefore, (8, 2.40) is the answer

Simplify.
Remove all perfect squares from inside the square root.
V63 =

I need the answer ASAP can anyone help?

Answers

Answer: [tex]3\sqrt{7}[/tex]

3*sqrt(7)

3 times the square root of 7

====================================================

Explanation:

I'm assuming the V stands for square root. You can write sqrt(63).

[tex]\sqrt{63} = \sqrt{9*7}\\\\\sqrt{63} = \sqrt{9}*\sqrt{7}\\\\\sqrt{63} = 3\sqrt{7}[/tex]

The idea is to factor 63 in such a way that one factor is the largest perfect square possible, that way we can pull the root apart to simplify as shown above. The rule I used for the second step is [tex]\sqrt{x*y} = \sqrt{x}*\sqrt{y}[/tex]

Match each expression with its greatest common factor. 4a + 8 2a2 + 8a 12a2 − 8a 4 − 6a Greatest Common Factor Expression 4 : 2 : 2a : 4a :

Answers

Answer:

see explanation

Step-by-step explanation:

4a + 8               4

2a² + 8a            2a

12a² - 8a           4a

4 - 6a                2

Match each expression with its greatest common factor as follows;

4a + 8               4

2a² + 8a            2a

12a² - 8a           4a

4 - 6a                2

What is the greatest common factor?

The largest number that is found in the common factors is called the greatest common factor.

The given expression are;

4a + 8              

2a² + 8a            

12a² - 8a      

4 - 6a              

The greatest common factor of the expression are as follows;

4a + 8  = 4(a+2) = common factor = 4    

2a² + 8a = 2a(a+4a) = common factor = 2a

12a² - 8a = 4a(3a-2) = common factor = 4a

4 - 6a = 2(2-3a) = common factor = 2

Learn more about the greatest common factor here;

https://brainly.com/question/15333869

#SPJ2

             

URGENT! What is the period for the cotangent function? (Answer in radians.)

Answers

it is [tex]\pi[/tex]

since period of tangent is $\pi$ ,so the period of reciprocal will also be same

The period for the cotangent function is π radians.

How do I find the period of a function?

The period for function y = A sin(Bx + C) and y = A cos(Bx + C) is 2π/|B| radians. Frequency is described as the wide variety of cycles completed in one 2d. If the duration of a function is denoted via P and f is its frequency, then –f =1/ P.

What is the period of tangent and cotangent?

The tangent function has period π. f(x)=Atan(Bx−C)+D is a tangent with vertical and/or horizontal stretch/compression and shift. The cotangent function has period π and vertical asymptotes at 0,±π,±2π, The range of cotangent is (−∞,∞), and the function is decreasing at each point in its range.

Learn more about the Cotangent function here https://brainly.com/question/2263992

#SPJ2

Complete the sequence
8,27,64,125,.........
Just next letter

Answers

Answer:

216.

Step-by-step explanation:

These numbers are perfect cubes starting with 2^3.

2^3, 3^3,  4^3,  5^3 so the next one is 6^3, which is 216.

A student finds the slope of the line between (8,17,) and (1,4) she writes 17-4/1-8 What mistake did she make?

Answers

The first mistake is that she didn't use parenthesis to indicate we're dividing all of one thing (numerator) over all of another expression (denominator)

slope = rise/run = numerator/denominator

So she should write (17-4)/(1-8) to indicate all of "17-4" is divided over all of "1-8"

However, there's another error that your teacher is probably more focused on. Note how 17-4 represents subtracting the y coordinates from left to right. We start with the left point y coordinate 17 and subtract off the right point y coordinate 4

left y - right y = 17 - 4

But then the student swaps the order when they wrote 1-8. Instead it should be 8-1

-----------------

Here's what they should have written (17-4)/(8-1)

This is the same as (4-17)/(1-8)

We can subtract in any order as long as it stays consistent between the numerator and denominator

Which of the c-values satisfy the following inequality? 2>c/3

Answers

Answer:

[tex]\Large \boxed{{c<6}}[/tex]

Step-by-step explanation:

2>c/3

Multiply both sides of the inequality by 3.

2(3)>c/3(3)

6>c

Switch sides.

c<6

●✴︎✴︎✴︎✴︎✴︎✴︎✴︎✴︎❀✴︎✴︎✴︎✴︎✴︎✴︎✴︎✴︎✴︎●

Hi my lil bunny!

❧⎯⎯⎯⎯⎯⎯⎯⎯⎯⎯⎯⎯⎯⎯⎯⎯⎯⎯⎯⎯⎯⎯⎯⎯⎯⎯⎯⎯⎯⎯⎯⎯⎯⎯⎯⎯⎯⎯☙

Let's solve your inequality step-by-step.

[tex]2 > \frac{c}{3}[/tex]

Step 1: Simplify both sides of the inequality.

[tex]2 > \frac{1}{3}c[/tex]

Step 2: Flip the equation.

[tex]\frac{1}{3}c < 2[/tex]

Step 3: Multiply both sides by 3.

[tex]3 * (\frac{1}{3} c) < ( 3) * (2)\\c < 6[/tex]

Answer : [tex]\boxed {c < 6}[/tex]

❧⎯⎯⎯⎯⎯⎯⎯⎯⎯⎯⎯⎯⎯⎯⎯⎯⎯⎯⎯⎯⎯⎯⎯⎯⎯⎯⎯⎯⎯⎯⎯⎯⎯⎯⎯⎯⎯⎯☙

●✴︎✴︎✴︎✴︎✴︎✴︎✴︎✴︎❀✴︎✴︎✴︎✴︎✴︎✴︎✴︎✴︎✴︎●

Have a great day/night!

❀*May*❀

A triangle has one side that lies along the line y=1/4x and another that lies along the line y=-1/4x. Which of the following points could be a vertex of the triangle?

Answers

Answer:

We know that our triangle has one side along the line:

y = (1/4)*x

And other side along the line:

y = -(1/4)*x.

Now, we want to find the vertex.

And we know that the vertex is the point where the two sides conect, so the vertex must be a common point of both lines.

Then we have:

y = (1/4)*x = -(1/4)*x

x = -x

The only solution to that equation is x = 0.

now we evaluate our lines in x = 0 and get:

y = (1/4)*0 = 0

y = -(1/4)*0 = 0

Then the lines intersect in the point (0, 0)

Then the vertex must be in the point (0, 0)

it’s either A or B but idk, i’ll give brainliest :)

Answers

Answer:

Actually it's A.

Step-by-step explanation:

Sofia ordered sushi for a company meeting. They change plans and increase how many people will be at the
meeting, so they need at least 100 pieces of sushi in total.
Sofia had already ordered and paid for 24 pieces of sushi, so she needs to order additional sushi. The sushi
comes in rolls, and each roll contains 12 pieces and costs $8.
Let R represent the number of additional rolls that Sofia orders.
1) Which inequality describes this scenario?
Choose 1 answer:

Answers

Answer:

The answer is given below

Step-by-step explanation:

The options are not given but I would list the inequalities needed to solve this problem.

As a result of increase in people to attend meeting, the number of sushi needed to be ordered is 100 pieces. Sofia has already ordered 24 pieces. If R is the number of additional rolls that Sofia orders.

The number of additional rolls that Sofia orders (R) must be greater than or equal to the difference between the number of Sushi needed to be ordered and the number of Sushi that has already being ordered. It is given by the inequality:

R ≥ 100 - 24

R ≥ 76

If each roll contains 12 pieces, the number of rolls needed (n) is given by:

n ≥ R/12

n ≥ 76/12

n ≥ 6.33

n ≥ 7

If each roll cost $8, the money needed to buy the sushi is given as:

Cost ≥ 8(7) ≥ $56

A number rounded to the nearest hundred thousand is 400,00. The same number rounded to the nearest ten thousand is 350,000. What could be the number?

Answers

Answer:

the number could be between 350,000 and 354,999

Step-by-step explanation:

when calculating to the nearest hundred thousand

for instance, your number is in this format abc,def, the hundred thousand place is determined by "b".

So if the value of b is over 5, when approximating, 1 is added to a.

if the value of b is below 5, when approximating, 0 is added to a.

Therefore, 354,999 to the nearest hundred thousand is 400,000

when calculating to the nearest ten thousand

for instance, your number is in this format abc,def, the ten thousand place is determined by "c".

So if the value of c is over 5, when approximating, 1 is added to b.

if the value of c is below 5, when approximating, 0 is added to b.

Therefore, 354,999 to the nearest ten thousand is 350,000

interpret the parts of the expression 9x + 4y – 5. Rewrite the expression as a sum: _______________

Answers

Answer:

9x + 4y +(-5)

Step-by-step explanation:

Given

9x + 4y - 5

Required

Interpret

Write as a sum

The parts of an expression can be interpreted in the following ways; Terms, Variables, Constant, Coefficient, etc.

The terms are the expression being added together and they are 9x, 4y and -5

The variables are the represented with alphabets they change in values; the two variables in the given question are x and y

Constant are numbers standing alone; This is 5

Coefficient are numbers in front of variables; In this case, the coefficient are 9 and 4

Writing 9x + 4y - 5 as a sum

The -5 can be written as +(-5); So, we have

9x + 4y +(-5)

If sin∅+cos∅ = 1 , find sin∅.cos∅.​

Answers

Answer:  0

=============================================

Explanation:

The original equation is in the form a+b = 1, where

a = sin(theta)

b = cos(theta)

Square both sides of a+b = 1 to get

(a+b)^2 = 1^2

a^2+2ab+b^2 = 1

(a^2+b^2)+2ab = 1

From here notice that a^2+b^2 is sin^2+cos^2 = 1, which is the pythagorean trig identity. So we go from (a^2+b^2)+2ab = 1 to 1+2ab = 1 to 2ab = 0 to ab = 0

Therefore,

sin(theta)*cos(theta) = 0

Answer:

sin ∅ cos ∅ = 0.

Step-by-step explanation:

(sin∅+cos∅)^2 = 1^2 = 1

(sin∅+cos∅)^2  = sin^2∅ + cos^2∅ + 2sin ∅ cos ∅ = 1

But  sin^2∅ + cos^2∅ = 1, so:  

2sin ∅ cos ∅ + 1 = 1

2 sin ∅ cos ∅ = 1 - 1 = 0

sin ∅ cos ∅ = 0.

Pluto is 2.7 times 10 to the power of 9 miles from the sun. Venus is 6.7 times 10 to the power of 7 miles from the sun. How mnay times greater is the distance of pluto to the sun than venus

Answers

Answer:here for the comments

Step-by-step explanation:

Answer:

The answer is 40.3 times greater

Step-by-step explanation:

Samuel has $20 in his savings account before he makes a deposit of $160 after two weeks he withdraws $160 how did Samuel savings account balance change

Answers

Samuel’s saving account changed from 20 to 180 to back to 20

Approximate 0.007349 to 3 significant figures​

Answers

Answer:

Hey there!

That would result in 0.00735.

Let me know if this helps :)

Hence, the significant figures are [tex]0.00735.[/tex]

What is the approximation?

An approximation is anything that is similar, but not exactly equal, to something else. A number can be approximated by rounding. A calculation can be approximated by rounding the values within it before performing the operations.

Here given that,

Approximate [tex]0.007349[/tex] to  significant figures​

So, the significant figures are [tex]0.00735.[/tex]

Hence, the significant figures are [tex]0.00735.[/tex]

To know more about the approximation

https://brainly.com/question/29580932

#SPJ2

Solve for x help please

Answers

Answer:

A. 5

Step-by-step explanation:

Based on the secant and tangent theorem, (4 + x)*4 = 6²

We can solve for x using the equation which describes the relationship between secant and tangents.

Thus,

[tex] (4 + x)*4 = 6^2 [/tex]

[tex] 4*4 + x*4 = 36 [/tex]

[tex] 16 + 4x = 36 [/tex]

Subtract 16 from both sides

[tex] 4x = 36 - 16 [/tex]

[tex] 4x = 20 [/tex]

Divide both sides by 4

[tex] \frac{4x}{4} = \frac{20}{4} [/tex]

[tex] x = 5 [/tex]

15 x = -27 + 6x cuál es la respuesta de la ecuación, ¿me ayudan? gracias!

Answers

Answer:

x = -3

Step-by-step explanation:

If the equation is correctly written as 15·x = -27 + 6·x

Then we have;

15·x = -27 + 6·x

We subtract 6·x from both sides to have all the variables (x) on one side as follows;

15·x - 6·x= -27 + 6·x - 6·x

Which gives;

x·(15 - 6) = -27 + x·(6 - 6)

We note that 6 - 6 = 0, therefore, we have;

x × 9 = -27 + x × 0

We not that x × 0 = 0, therefore;

x × 9 = -27

We now divide both sides with the 9 to get the value of x alone as follows;

x×9/9 = -27/9

Which gives;

x = -27/9 = -3

x = -3

20 POINTS AND BRAINLIEST!! Which of the following statements can be supported by the evidence shown in the graph?

A. The line best fit shows a positive correlation between month number and minutes

B. The line of best fits shows a negative correlation between the number of months and the number of minutes

C. The line of best fits shows no correlation between the month number and the number of minutes

Answers

Answer:

B. The line of best fits shows a negative correlation between the number of months and the number of minutes

Step-by-step explanation:

In a scatter plot, when the data points are clustered along the line of best fit, it shows a trend, and it implies there's a correlation between the x-variable and the y-varisble.

Also, when the line of best fit slopes upward, from your left to your far right, it shows a positive correlation between the variables on the x-axis and y-axis.

On the other hand, if the line of best fit slopes downwards, from your left down to your right, this shows a negative correlation.

Thus, considering the line of best fit in the question, the line of best fit shows a negative correlation between the number of months and the number of minutes. The line of best fit slopes downwards, from your left to your right.

This shows that, as month number increases, number of minutes decreases.

Which of the binomials below is a factor of this trinomial? URGENT!!!

Answers

Answer:

C

Step-by-step explanation:

10×-28=-280

35-8=27

35×(-8)=-280

10x²+27x-28

=10x²+(35-8) x-28

=10x²+35x-8x-28

=5x(2x+7)-4(2x+7)

=(2x+7)(5x-4)

Answer: C) 2x+7

=========================================================

Explanation:

One way we can factor is through use the of the quadratic formula.

Let [tex]10x^2+27x-28 = 0[/tex]

For now, the goal is to find the two roots of that equation.

Plug a = 10, b = 27, c = -28 into the quadratic formula

[tex]x = \frac{-b\pm\sqrt{b^2-4ac}}{2a}\\\\x = \frac{-(27)\pm\sqrt{(27)^2-4(10)(-28)}}{2(10)}\\\\x = \frac{-27\pm\sqrt{1849}}{20}\\\\x = \frac{-27\pm43}{20}\\\\x = \frac{-27+43}{20} \ \text{ or } \ x = \frac{-27-43}{20}\\\\x = \frac{16}{20} \ \text{ or } \ x = \frac{-70}{20}\\\\x = \frac{4}{5} \ \text{ or } \ x = -\frac{7}{2}\\\\[/tex]

The two roots are x = 4/5 and x = -7/2

For each root, rearrange the equation so we have 0 on the right hand side, and it's ideal to get rid of the fractions

x = 4/5

5x = 4

5x-4 = 0 gives us one factor

and

x = -7/2

2x = -7

2x+7 = 0 gives the other factor

The two factors are 5x-4 and 2x+7

Note how (5x-4)(2x+7) = 0 leads to the two separate equations of 5x-4 = 0 and 2x+7 = 0 due to the zero product property. Solving each individual equation leads to the two roots we found earlier.

Alternative methods to solve this problem are the AC factoring method (which leads to factor by grouping), using the box method, or you could use guess and check.

SOMEONE PLZ HELP ASAP !!!!!!

Answers

Answer:

a = 53.13

b = 42.76

c= 0

Step-by-step explanation:

as the question said i was supposed to use a calculator which i did

Write each of the following in the simplest exponential form please help

Answers

Answer:

Step-by-step explanation:

[tex]a^{m}*a^{n} =a^{m+n}\\\\\\2^{3}*2^{5} = 2^{3+5} =2^{8}\\\\\\3^{2}*3^{4}=3^{2+4} = 3^{6}\\\\\\a^{2}*a^{5}=a^{2+5}=a^{7}[/tex]

A company is making a new label for one of their containers. The container is a cylinder that is 9 inches tall and 5 inches in diameter. What is the area of the label that needs to be printed to go around the new container? Use π = 3.14.

Answers

Answer:

180.55 in².

Step-by-step explanation:

Data obtained from the question include the following:

Height (h) = 9 in.

Diameter (d) = 5 in

Pi (π) = 3.14

Area of the label =..?

Next, we shall determine the radius.

Diameter (d) = 5 in

Radius (r) =.. ?

Radius (r) = Diameter (d) /2

r = d/2

r = 5/2

r = 2.5 in.

Next, we shall determine the area of the label that needs to be printed to go around the new container by calculating the surface area of the cylinder.

This is illustrated below:

Height (h) = 9 in.

Pi (π) = 3.14

Radius (r) = 2.5 in.

Surface Area (SA) =.?

SA = 2πrh + 2πr²

SA = (2×3.14×2.5×9) + (2×3.14×2.5²)

SA = 141.3 + 39.25

SA = 180.55 in²

The surface area of the cylinder is 180.55 in².

Therefore, the area of the label that needs to be printed to go around the new container is 180.55 in².

Answer:

Step-by-step explanation:

i dont have the work but the answer is 182.6

Other Questions
What are Red Bull's greatest strengths and risks as more companies (like Coca-Cola, Pepsi, and Monster) enter the energy drink category and gain market share? Sonic Inc. manufactures two models of speakers, Rumble and Thunder. Based on the following production and sales data for June, prepare (a) a sales budget and (b) a production budget: Rumble Thunder Estimated inventory (units), June 1 750 300 Desired inventory (units), June 30 500 250 Expected sales volume (units): Midwest Region 12,000 3,500 South Region 14,000 4,000 Unit sales price $60 $90 a. Prepare a sales budget. On January 1, a company issues bonds dated January 1 with a par value of $320,000. The bonds mature in 5 years. The contract rate is 7%, and interest is paid semiannually on June 30 and December 31. The market rate is 6% and the bonds are sold for $333,650. The journal entry to record the first interest payment using the effective interest method of amortization is: (Rounded to the nearest dollar.) Many managers describe performance appraisal as the responsibility that they like least. Why is this so? What could be done to improve the situation? In May 1974, the U.S. government initiated the Federal Design Improvement Program in response to a growing awareness of design as an effective tool for achieving objectives. One of the most successful federal visual identification systems was the Unigrid system, a standardized format developed in 1977 for the _______________. Your supervisor instructs you to purchase 480 pens and 6 staplers for the workplace. Pens are purchased in sets of 6 for $2.45. Staplers are sold in sets of 2 for $14.95. How much will the purchase of these products cost? A division of a manufacturing company has a return on investment of 24%. The division has an opportunity to accept a project that is expected to earn a return on investment of 22%. The companys hurdle rate is 20% which of the following statements is true? a) A division reports the following figures: Profit margin =20% Investment turnover = 0.5. The division return on investment is b) If a company has $2,000,000 invested in buildings, equipment, and other assets and desires to earn a return on investment of 30%, the company will need to earn a net income of $ . how many cars the baseball team needs to wash before it starts making a profit. The team spent $75 setting up the car wash, and they are charging $5 per car for a wa The first step in modeling this situation is to track how much money the baseball team will take in. Write an equation to represent the amount of money collected in dollars, y, in terms of the number of cars washed, x. Ignore the setup cost. WILL MARK BRAINLIEST FOR CORRECT ANSWER. Let u = , v = . Find u + v. (1 point) What are thetraditionalcirculatoryendpoints ofresuscitationin shock? Please make 6 Sentences usingSo..... ThatSuch.... That A project requires an investment of $10 million and offers an annual after-tax cash flow of $1,250,000 indefinitely. If the firm's WACC is 12.5% and the project is riskier than the firm's average projects, should it be accepted? Given these four points: A(3, 3), B(5, 7), C(2, 11), and D(9, 2), find the coordinates of the midpoint of line segments AB and CD. On which point do Freud and current developmental psychologists disagree? Group of answer choices importance of early experiences idea that human behavior can be influenced by unconscious or implicit processes importance of close relationships the importance of psychosexual drives The numbers in the select boxes are 4 7 and 3 I put them with the photos. Can someone help Puzzle cornerLook Before You Leap!See how long it takes you to work out thefollowing:(1 x2)(3 x 4)(586)(7 x 8) x (90) Which word group is a complete sentence?Started his homework late Sunday night.Two energetic women walking in the park.She did not understand what happened.The exciting end of the baseball game. Julio wants to solve the system shown using the elimination method. Which is the best way to begin?(x - 12y = 2-4x + 7y = 12Add the equationsb. Multiply each term in x - 12y = 2 by 4 and add it to the other original equation.This system of equations has no solution, so Julio should not do anything.d. Multiply each term in x - 12y = 2 by 4 and add it to the other original equation.c. estimate the number 4576